1987 AJHSME Problems/Problem 3

Revision as of 14:47, 29 January 2009 by Misof (talk | contribs)

Problem

$2(81+83+85+87+89+91+93+95+97+99)=$

$\text{(A)}\ 1600 \qquad \text{(B)}\ 1650 \qquad \text{(C)}\ 1700 \qquad \text{(D)}\ 1750 \qquad \text{(E)}\ 1800$

Solution

$2(81+83+85+87+89+91+93+95+97+99)$

$\begin{align*} & = 2\Big( (81+99) + (83+97) + (85+95) + (87+93) + (89+91) \Big) \\ & = 2(180+180+180+180+180) \\ & = 2\cdot 5\cdot 180 \\ & = \boxed{1800}. \end{align*}$ (Error compiling LaTeX. Unknown error_msg)

See Also

1987 AJHSME Problems